Edits
This commit is contained in:
parent
689d085e6f
commit
56b72a0531
@ -17,8 +17,7 @@
|
|||||||
% use the [solutions] flag to show solutions.
|
% use the [solutions] flag to show solutions.
|
||||||
\documentclass[
|
\documentclass[
|
||||||
solutions,
|
solutions,
|
||||||
singlenumbering,
|
singlenumbering
|
||||||
unfinished
|
|
||||||
]{../../resources/ormc_handout}
|
]{../../resources/ormc_handout}
|
||||||
\usepackage{../../resources/macros}
|
\usepackage{../../resources/macros}
|
||||||
|
|
||||||
@ -46,53 +45,25 @@
|
|||||||
\input{parts/03.00 logic gates}
|
\input{parts/03.00 logic gates}
|
||||||
\input{parts/03.01 quantum gates}
|
\input{parts/03.01 quantum gates}
|
||||||
|
|
||||||
|
%\section{Superdense Coding}
|
||||||
|
%TODO
|
||||||
|
%\vfill
|
||||||
|
%\pagebreak
|
||||||
|
|
||||||
|
%\section{Quantum Error Correction}
|
||||||
|
%TODO: no cloning theorem, bit flip code
|
||||||
|
%\vfill
|
||||||
|
%\pagebreak
|
||||||
|
|
||||||
|
|
||||||
|
%\section{Quantum Teleportation}
|
||||||
|
%TODO
|
||||||
|
%\vfill
|
||||||
|
%\pagebreak
|
||||||
|
|
||||||
|
%\section{One Real Qubit}
|
||||||
|
% No problems, appendix.
|
||||||
|
% bloch sphere, etc.
|
||||||
|
|
||||||
|
|
||||||
\end{document}
|
\end{document}
|
||||||
|
|
||||||
\problem{}
|
|
||||||
The SWAP gate swaps two bits: $\text{SWAP}\ket{ab} = \ket{ba}$. \par
|
|
||||||
Find its matrix.
|
|
||||||
|
|
||||||
\begin{solution}
|
|
||||||
\begin{equation*}
|
|
||||||
\text{SWAP} = \begin{bmatrix}
|
|
||||||
1 & 0 & 0 & 0 \\
|
|
||||||
0 & 0 & 1 & 0 \\
|
|
||||||
0 & 1 & 0 & 0 \\
|
|
||||||
0 & 0 & 0 & 1 \\
|
|
||||||
\end{bmatrix}
|
|
||||||
\end{equation*}
|
|
||||||
\end{solution}
|
|
||||||
|
|
||||||
\vfill
|
|
||||||
|
|
||||||
|
|
||||||
|
|
||||||
|
|
||||||
|
|
||||||
|
|
||||||
|
|
||||||
|
|
||||||
|
|
||||||
% \problem{}
|
|
||||||
% The $T$ gate is a three-bit gate that inverts its right bit iff its left and middle inputs are both $\ket{1}$. \par
|
|
||||||
% In other words, $T\ket{11x} = \ket{11}\ket{\text{not } x}$, and $T\ket{abx} = \ket{abx}$ for all other inputs. \par
|
|
||||||
% Find the $T$ gate's matrix. \par
|
|
||||||
% \note{
|
|
||||||
% This gate is particularly interesting because it's a \textit{universal quantum gate}: \\
|
|
||||||
% like NOR and NAND in classical logic, any quantum gate may emulated by only applying $T$ gates.
|
|
||||||
% }
|
|
||||||
%
|
|
||||||
% \begin{solution}
|
|
||||||
% \begin{equation*}
|
|
||||||
% \text{T} = \begin{bmatrix}
|
|
||||||
% 1 & 0 & 0 & 0 & 0 & 0 & 0 & 0 \\
|
|
||||||
% 0 & 1 & 0 & 0 & 0 & 0 & 0 & 0 \\
|
|
||||||
% 0 & 0 & 1 & 0 & 0 & 0 & 0 & 0 \\
|
|
||||||
% 0 & 0 & 0 & 1 & 0 & 0 & 0 & 0 \\
|
|
||||||
% 0 & 0 & 0 & 0 & 1 & 0 & 0 & 0 \\
|
|
||||||
% 0 & 0 & 0 & 0 & 0 & 1 & 0 & 0 \\
|
|
||||||
% 0 & 0 & 0 & 0 & 0 & 0 & 0 & 1 \\
|
|
||||||
% 0 & 0 & 0 & 0 & 0 & 0 & 1 & 0 \\
|
|
||||||
% \end{bmatrix}
|
|
||||||
% \end{equation*}
|
|
||||||
% \end{solution}
|
|
@ -290,8 +290,7 @@ We could draw the above transformation as a combination $X$ and $I$ (identity) g
|
|||||||
\end{tikzpicture}
|
\end{tikzpicture}
|
||||||
\end{center}
|
\end{center}
|
||||||
|
|
||||||
We can even omit the $I$ gate, since we now know that transforms affect the whole state. \par
|
We can even omit the $I$ gate, since we now know that transforms affect the whole state: \par
|
||||||
Of course, empty spaces always imply an $I$ gate.
|
|
||||||
\begin{center}
|
\begin{center}
|
||||||
\begin{tikzpicture}[scale=0.8]
|
\begin{tikzpicture}[scale=0.8]
|
||||||
\node[qubit] (a) at (0, 0) {$\ket{0}$};
|
\node[qubit] (a) at (0, 0) {$\ket{0}$};
|
||||||
|
@ -105,9 +105,8 @@ Find the matrix that applies the cnot gate.
|
|||||||
\vfill
|
\vfill
|
||||||
|
|
||||||
\generic{Remark:}
|
\generic{Remark:}
|
||||||
Note that a quantum gate is fully defined by the place it maps
|
As we just saw, a quantum gate is fully defined by the place it maps our basis states $\ket{0}$ and $\ket{1}$ \par
|
||||||
our basis states $\ket{0}$ and $\ket{1}$ (or,$\ket{00...0}$ through $\ket{11...1}$ for multi-qubit gates).
|
(or, $\ket{00...0}$ through $\ket{11...1}$ for multi-qubit gates). This directly follows from \ref{qgateislinear}.
|
||||||
This directly follows from \ref{qgateislinear}.
|
|
||||||
|
|
||||||
|
|
||||||
\pagebreak
|
\pagebreak
|
||||||
@ -172,28 +171,18 @@ If we measure the result of \ref{applycnot}, what are the probabilities of getti
|
|||||||
%\vfill
|
%\vfill
|
||||||
|
|
||||||
|
|
||||||
|
|
||||||
\definition{}
|
\definition{}
|
||||||
The \textit{Hadamard Gate} $H$, is given by the following matrix: \par
|
The \textit{Hadamard Gate} is given by the following matrix: \par
|
||||||
\begin{equation*}
|
\begin{equation*}
|
||||||
H = \frac{1}{\sqrt{2}}\begin{bmatrix}
|
H = \frac{1}{\sqrt{2}}\begin{bmatrix}
|
||||||
1 & 1 \\
|
1 & 1 \\
|
||||||
1 & -1
|
1 & -1
|
||||||
\end{bmatrix}
|
\end{bmatrix}
|
||||||
\end{equation*}
|
\end{equation*}
|
||||||
\note[Note]{Note that we divide by $\sqrt{2}$, since $H$ must be orthonormal}
|
\note{Note that we divide by $\sqrt{2}$, since $H$ must be orthonormal.}
|
||||||
|
|
||||||
|
\begin{ORMCbox}{Review: Matrix Multiplication}{black!10!white}{black!65!white}
|
||||||
\problem{}
|
|
||||||
What is $HH$? \par
|
|
||||||
Using this result, find $H^{-1}$.
|
|
||||||
|
|
||||||
\begin{solution}
|
|
||||||
$HH = I$, so $H^{-1} = H$
|
|
||||||
\end{solution}
|
|
||||||
|
|
||||||
\vfill
|
|
||||||
|
|
||||||
\begin{ORMCbox}{Review: Matrix Multiplication}{black!10!white}{black}
|
|
||||||
Matrix multiplication works as follows:
|
Matrix multiplication works as follows:
|
||||||
|
|
||||||
\begin{equation*}
|
\begin{equation*}
|
||||||
@ -236,4 +225,25 @@ Using this result, find $H^{-1}$.
|
|||||||
This is exactly the first column of the matrix product.
|
This is exactly the first column of the matrix product.
|
||||||
\end{ORMCbox}
|
\end{ORMCbox}
|
||||||
|
|
||||||
|
|
||||||
|
\problem{}
|
||||||
|
What is $HH$? \par
|
||||||
|
Using this result, find $H^{-1}$.
|
||||||
|
|
||||||
|
\begin{solution}
|
||||||
|
$HH = I$, so $H^{-1} = H$
|
||||||
|
\end{solution}
|
||||||
|
|
||||||
|
\vfill
|
||||||
|
|
||||||
|
\problem{}
|
||||||
|
What are $H\ket{0}$ and $H\ket{1}$? \par
|
||||||
|
Are these states superpositions?
|
||||||
|
|
||||||
|
\begin{solution}
|
||||||
|
$H\ket{0} = \frac{1}{\sqrt{2}}\bigl(\ket{0} + \ket{1}\bigr)$ and $H\ket{1} = \frac{1}{\sqrt{2}}\bigl(\ket{0} - \ket{1}\bigr)$ \par
|
||||||
|
Both of these are superpositions.
|
||||||
|
\end{solution}
|
||||||
|
|
||||||
|
\vfill
|
||||||
\pagebreak
|
\pagebreak
|
||||||
|
Loading…
x
Reference in New Issue
Block a user